UTNianos

Versión completa: Ayuda- Integrales Multiples
Actualmente estas viendo una versión simplificada de nuestro contenido. Ver la versión completa con el formato correcto.
[attachment=1539]Buenas gente, aca dejo a ver si me pueden dar una mano con este ejercicio q me esta volviendo loco, es el ejercicio 7 de la guia de AM2 unidad 9
Lo intento hacer con coordenadas polares y no llego al resultado, y lo ultimo que intente fue sacar areas por separado y hacer la diferencia.
Hago otra consulta, cuando realizo coordenadas polares, se aplica el determinande de la funcion? no se si me explico..
Gracias gente!!!
tenes que multiplicar por "ro" si esa es tu pregunta.

otra cosa, pensaste que tal vez la proyeccion no da un recinto circular, por lo que no podes aplicar coordenadas polares ?

la verdad que no tengo tiempo para hacerlo, pero fijate eso sino, proba con las cartesianas
saludos
Mira, hice con las cartesiana para ver si encontraba otra manera, es decir, area por area, y me da un valor diferente cn el de la guia, con las polares intente, pero como no es un recinto en forma de circunferencia y con la parametrizacion se me complico,tampoco llego al resultado...la vdd q no lo encuentro la forma o estare planteando algo mal seguro..
Con el tema de multiplicar por "ro" si lo hice, pero en algunos ejercicios, vi que el profe multiplica dsp por el det del la jacobiana.. tngo una confusion me parece terrible..
sigo intentando mientras a ver si lo puedo sacar
el determinante de la matriz jacobiana varia segun las coordenadas que apliques

en cartesianas ni siquiera existe porque no hay cambio

en polares es \[\rho\]

en cilindricas es \[\rho\]

en elipticas es \[\rho a.b\]

a y b provienen de

\[x= a.\rho\cos \lambda\\y= b.\rho\sin \lambda\]

en esfericas es \[\rho^2 \sin\theta\]

donde \[z=\rho \cos\theta\]

edit: escribi mal "\[\theta\]"
Hola, segui la sugerencia del el pibe y aplica la expresión polar de la elipse usando coordenadas

\[g(r,\theta)=(ar\cos\theta,br\sin\theta)\]

siendo a y b el semidiametro mayor y menor de la elipse, al hacerlo no te olvides poner todos los nuevos límites en funcion de las nuevas coordenadas, si querés y podés, postea lo que hiciste para ver en que te estas equivocando, eso si queres y podes obviamente

saludos

Off-topic:
Una consulta El pibe es el mismo que el usuario del foro de matemáticas??!
(01-10-2011 21:48)Feer escribió: [ -> ]
Off-topic:
Una consulta El pibe es el mismo que el usuario del foro de matemáticas??!


Off-topic:
¿que foro? rincon??

Off-topic:
Sip, por ahí mire cualquiera
(01-10-2011 22:05)Feer escribió: [ -> ]
Off-topic:
Sip, por ahí mire cualquiera


Off-topic:
puede ser, el mismo , y en rincon vos sos???
(01-10-2011 22:11)aoleonsr escribió: [ -> ]
(01-10-2011 22:05)Feer escribió: [ -> ]
Off-topic:
Sip, por ahí mire cualquiera


Off-topic:
puede ser, el mismo , y en rincon vos sos???


Off-topic:
Entre hace 3 días porque me intereso la parte de teoría del caos que vi y me quede jajaja, esta bueno y ya me instale:O, me llamo feer también ahí, pero como mucho hice 7 post.
(01-10-2011 22:17)Feer escribió: [ -> ]
Off-topic:
Entre hace 3 días porque me intereso la parte de teoría del caos que vi y me quede jajaja, esta bueno y ya me instale:O, me llamo feer también ahí, pero como mucho hice 7 post.


Off-topic:
bienvenido a ese foro thumbup3 =D =D

Off-topic:
no se de que foro hablan, estoy en muchos (?)
si se que en algun lado hable/discuti sobre la teoria del caos, pero me parece que no soy ese de quien hablan, porque no recuerdo ningun foro de matematica xD
Bueno antes que nada gracias por los datos para poder resolverlo.. Te cuento que hice extactamente eso, aplicado coordenadas polares, y multiplicando por "ro", pero me parece que me estoy confundiendo con los limites de integracion.
Dejame ver si puedo subir lo que estoy planteando y vemos en que le estoy pifiando.. gracias viejo!!!
acordate que los (x^2 + y^2) se transforman en ro^2
URLs de referencia